Unendliches Widerstandsnetzwerk

Hallo,

Neulich versuchte ich den Gesamtwiderstand folgendes Widerstandsnetzwerkes zu berechnen:

___ ___ ___ --|___|--+--|___|--+--|___|--+---.... unendlich fortgesetzt | | | .-. .-. .-. | | | | | | | | | | | | '-' '-' '-' | | | ---------+---------+---------+---....

Die vertikalen Widerstände haben alle einen Wert Rv und die horizontalen einen Wert von Rh. Es sollte sich einen Ausdruck für den Gesamtwiderstand ergeben der von n abhängt, und der für n-> unendlich einen Grenzwert irgendwo zwischen Rh und Rh+Rv hat.

Aber wie ich es auch anstelle ich kriege keinen geschlossenen Ausdruck hin, sondern eine furchtbare Formel die für jedes zusätzliche n immer größer und komplexer wird. Dabei habe ich es auch über Kettenmatrizen der Vierpoldarstellung versucht. Aber jedesmal werden die Ausdrücke immer komplexer und ich kann kein Bildungsgesetz finden.

Dabei hat das Problem Ahnlichkeit mit der Berechnung des Wellenwiderstandes einer unendlich langen Leitung, wobei Kapazitäten und Induktivitäten dabei sind. Leider hab ich nirgendwo, weder im Internet, noch in Fachbüchern eine ausreichend detailierte Herleitung desselbigen gefunden, die mir helfen könnte ein Ergebnis für das obige Problem zu finden.

Wenn mir jemand eine Referenz zu dem Wellenwiderstand geben könnte wäre mir schon geholfen..

Danke, Mfg Moritz

Reply to
moritz erbs_löh
Loading thread data ...

"moritz erbs_löh"

berechnen:

Wert von Rh.

abhängt, und der für n-> unendlich einen Grenzwert

Erstes Widerstandsglied Rh+Rv zweites Rh+Rh+Rv

=> Summe(n = 1 bis oo) (n*Rh+Rv)

diese sind dann alle Parallel.

Also Summe(n=1 bis oo) (1/[1/[Summe(n = 1 bis oo) (n*Rh+Rv)]+1/[Summe(n = 1 bis oo) (n*Rh+Rv)]])

Hmm... bin mir aber jetzt nicht sicher ob das wirklich richtig ist aber ist auf jeden Fall eine Interessante Übungsaufgabe

Reply to
Markus

kes=20

len=20

n n=20

n Rh und=20

Angenommen, es konvergiert wirklich. Dann kann man einen kleinen Trick anwenden: Der Gesamtwiederstand der unendlich langen Kette sei der Grenzwert Rx, dann =E4ndert sich nichts, wenn Du noch ein Rh/Rv-Glied davorh=E4ngst, also ist Rx =3D Rh+(Rv||Rx). Diese Gleichung l=E4sst sich nach Rx aufl=F6sen, hat aber mehr als eine L=F6sung, aber nur eine ist physikalisch sinnvoll und liegt im Bereich, den Moritz erwartet.

Michael Karcher

Reply to
Michael Karcher

Michael Karcher schrieb:

Sehr schön, wenn ich mich nicht vertan habe müsste da rauskommen: Rx = sqrt(Rh*Rv + Rh²/4) + Rh/2

Das mit den n Gliedern ist für Mathematiker sicherlich ein einfacher Integrations-Fall, evtll mal in d.s.m stellen?

Gruß Jens

Reply to
Jens Dierks

Jens Dierks schrieb:

Nix Integration, unendliche Reihe. Bronstein, Semendjajew et al.... :-)

Gruß Henning

--
henning paul home:  http://www.geocities.com/hennichodernich
PM: henningpaul@gmx.de , ICQ: 111044613
Reply to
Henning Paul

Kann man nicht jedes (Riemann-) Integral als unendliche Reihe und jede unendliche Reihe als (Riemann-) Integral auffassen?

Michael

Reply to
Michael J. Schülke

meinst Du hier Eingangs- und Ausgangsimpedanz des Vierpols? Ich erinnere mich dunkel an eine ähnliche Diskussion hier vor einiger Zeit. Mit dem Kettenleiteransatz würde ich es lieber nicht versuchen, das wird unübersichtlich.

Das mit den Kettenmatrizen halte ich für den richtigen Ansatz. Du brauchst nur eine einzige kleine 2 x 2 - Matrix, die Dir alles beschreibt.

Aber hör mal, das ist Grundlagenstoff im Studium E-Technik, Schwerpunkt Nachrichtentechnik, das lernt doch jeder. Bücher müßte es massenhaft dazu geben, weiß die heutzutage aktuellen jetzt nicht. Ich glaube nicht, daß das Internet zum Lernen geeignet ist. Heutzutage glaube viele, daß man im Internet fertige Lösungen für alles findet, ich halte eher das Gegenteil für richtig.

Zerlege einfach die unendliche Kette in einfache richtungssymmetrische reziproke Vierpole, der Wellenwiderstand läßt sich leicht bestimmen, mit dem mußt Du dann zusätzlich Ein- und Ausgang abschließen.

Bekanntester Sonderfall (binäre Abschwächung) wäre das R2R-Netzwerk, das in DA-Wandlern verwendet wird, das ist besonders übersichtlich, damit kannst Du's ja mal testen.

Wellenwiderstand eines symmetrischen Vierpols:

Zl=sqrt(ZB1l*ZB1K)=sqrt(A12/A21)

ZB1l= Eingangsimpedanz bei leerlaufendem Ausgang ZB1K= Eingangsimpedanz bei kurzgeschlossenem Ausgang A12, A21 Vierpol-Kettenparameter != 0

Ich würde es auf symmetrische Vierpole vereinfachen, auch für unsymmetrische habe ich hier fertige Gleichungen, bei Bedarf kann ich die posten. Solange Du das mit dem Wellenwiderstand abschließt, ist das IMHO nahezu trivial, bei Fehlanpassung jedoch nicht. Im Prinzip könnte man das als verlustbehaftete Leitung betrachten.

mfg. Winfried

Reply to
Winfried Salomon

Warum so kompliziert?

Wenn man davon ausgeht, dass es eine Lösung gibt, dann gilt:

Re = Rh + (Re || Rv)

Re || Rv = (Re*Rv) / (Re + Rv)

Re = Rh + (Re*Rv) / (Re + Rv)

Re : Eingangswiderstand Rv: vertikaler Widerstand Rh: horizontaler Widerstand

Gruß

Stefan

Reply to
Stefan Brroering

Ups, die Lösung stand schon weiter oben, tschuldigung

Reply to
Stefan Brroering

Wenn du den Integranden über dem Intrgrationsintervall in eine gleichmäßig konvergente Funktionenreihe entwickeln kannst, dann ja. Das sollte für analytische Funktionen erfüllt sein. Aber wie gesagt: Funktionen, nicht eine Zahlenfolge.

Das geschilderte Problem ist einfach durch unendlich+1=unendlich lösbar. Ein klassisches Problem, das Erstsemestrige bei uns immer vorgesetzt bekommen und das ein echter Klassiker ist.

Die Lösung wurde bereits angegeben.

Für eine endliche Struktur gibt es die Theorie der Kontinuanten. Sollte man sich einmal ansehen. Das sind Bidiagonal-Determinanten zur Darstelling eines Kettenbruches. In diesen kann man solche endliche Netze entwickeln

lg Johannes

Reply to
John F

John F schrieb:

Ich kenne den Ansatz bspw. von Markovketten, um Aufenthaltswahrscheinlichkeiten in bestimmten Zuständen zu bestimmen.

Gruß Henning (der Warteschlangentheorie nie sonderlich gemocht hat, es aber leider Pflichtfach war)

--
henning paul home:  http://www.geocities.com/hennichodernich
PM: henningpaul@gmx.de , ICQ: 111044613
Reply to
Henning Paul

Ja da geht das auch. Aber eben nur, wenn der Kettenbruch Konvergiert. Im Fall von Markov kommt die zeitliche Verteilung noch dazu.

-- lg Johannes

Reply to
John F

Genau das ist dein Problem. Du gehst die Sache aus der Richtung des ersten Gliedes an.

Du mußt das Problem von hinten, aus Richtung des n-ten Gliedes formulieren. Dann hast du nur noch zwei Terme, mit denen du dich rumschlagen mußt (Rpn=Teilwiderstand des n-ten Gliedes):

Rpn=Rh+Rv Rpn-1=Rh+Rv||Rpn

Wenn du's mit dieser Hilfe nicht rausbekommst, dann ziehe die Konsequenz und verschwende bitte nicht weiter meine Steuergelder.

Reply to
Heiko Nocon

moritz erbs_löh schrieb:

Mein Tipp:

formatting link

Das Problem ist aber, daß bei der Leitung ein Widerstands_belag_ angesetzt wird, das heißt, die absoluten Widerstandswerte gehen gegen 0, wenn die betrachteten Abschnitte immer kleiner werden.

Die Lösung für die transmission line würde lauten

Z_L=sqrt((R'+jomegaL')/(G'+jomegaC')), d.h. in Deinem Fall Z_L=sqrt(R'/G'). Nur leider ist R' hier nicht einfach durch Rv und G' durch 1/Rh zu ersetzen.

Gruß Henning

--
henning paul home:  http://www.geocities.com/hennichodernich
PM: henningpaul@gmx.de , ICQ: 111044613
Reply to
Henning Paul

=20

=20

Meiner Ansicht nach ergibt sich das unmittelbar aus der Definition des=20 Riemann-Integrals, d.h. als Grenzwert einer Folge von Summen von immer=20 feineren Treppenfunktionen; aus dieser Folge l=E4=DFt sich wiederum eine=20 unendliche Reihe konstruieren.=20

Andersrum l=E4=DFt sich zu jeder Reihe eine Treppenfunktion konstruieren,= =20 deren (uneigentliches) Integral (von 0 bis plus unendlich) der Grenzwert=20 der Reihe ist. =20

Michael

Reply to
Michael J. Schülke

"Michael J. Schülke" wrote in dse:

Die Zerlegung des Intervalls muss dadurc ausgezeichnet sein, dass der Grenzwert der Summe (von i=1 bis n) über f(z_i)*d x_{i-1} von der Wahl der z_i und x_{i-1} unabhängig ist und das erfüllen nicht alle Folgen x_i.

Hm. Müsste man in dsm fragen.

follow up gesetzt.

Mal sehn, was die Experten sagen.

lg Johannes A. Frittum

Reply to
John F

"Markus" schrieb im Newsbeitrag news:44522b53$0$4503$ snipped-for-privacy@newsread2.arcor-online.net...

Ich bin mir nicht Sicher wie du auf diese Lösung gekommen bist. Wieso z.B. Rh+Rh+Rv für das 2. Glied? In der Endformel verwendest du 3 Summenausdrücke, sollten die variablen dann nicht auch unterschiedlich sein?

Mfg Moritz

Reply to
moritz erbs_löh

"Jens Dierks" schrieb im Newsbeitrag news:44523ae2$0$5440$ snipped-for-privacy@news.freenet.de...

Ich habe dasselbe Ergebnis raus, das war ja sehr schön einfach und geschickt, aber ist es das tatsächlich? Ich hätte auch erwartet irgendwas integrieren zu müssen oder eine Grenzwertbetrachtung vornehmen zu müssen. Wie kommst du also auf n Glieder, jetzt da in dem Ergebnis kein n mehr auftaucht?

Mfg Moritz

Reply to
moritz erbs_löh

"Henning Paul" schrieb im Newsbeitrag news: snipped-for-privacy@fump.de.vu...

Welche unendliche Reihe ist gemeint? Das Ergebnis von Michael stellt doch schon eine geschlossene Formel da?

Mfg Moritz

Reply to
moritz erbs_löh

moritz erbs_löh schrieb:

Ich meinte, daß Du den Kettenbruch als Reihe darstellen kannst. Als welche, kann ich auch nicht sagen. :-)

Ja, ist auch der einfachste Lösungsweg.

Gruß Henning

--
henning paul home:  http://www.geocities.com/hennichodernich
PM: henningpaul@gmx.de , ICQ: 111044613
Reply to
Henning Paul

ElectronDepot website is not affiliated with any of the manufacturers or service providers discussed here. All logos and trade names are the property of their respective owners.